Two angles are suplementary with measueres m Angle: QRS = (x + 30) and m Angle: SRT = (2x) . What is the value of m Angle: QRS?

Since the two angles are supplementary, their measures add up to 180 degrees.

Therefore, we can set up the equation:

(x + 30) + (2x) = 180

Simplify:

3x + 30 = 180

Subtract 30 from both sides:

3x = 150

Divide by 3:

x = 50

Now, plug x back into the measure of angle QRS:

m Angle QRS = x + 30
m Angle QRS = 50 + 30
m Angle QRS = 80

Therefore, the measure of angle QRS is 80 degrees.

TYSM!! ;D

You're welcome! If you have any more questions or need further assistance, feel free to ask. Good luck! 😊

Four angles share a vertex. They have the following measures: m Angle 1 = 63

m angle 2 =117, m angle 3 = 63, and m angle 4 = 117. Which angle is a vertical angle with angle 2?

Vertical angles are the angles that share the same vertex and are formed by two intersecting lines.

In this case, angle 1 and angle 2 are vertical angles because they have the same vertex and are formed by the intersecting lines that make up angle 1 and angle 2.

Therefore, angle 1 is the vertical angle with angle 2.

For two vertical angles with measures m angle 2 = 135 and m angle 4 = (4x+15) . What is the value of x?

Vertical angles are always congruent, meaning they have the same measure.

Since angle 2 and angle 4 are vertical angles, their measures are equal:

m angle 2 = m angle 4
135 = 4x + 15

Now, solve for x:

4x + 15 = 135
4x = 120
x = 30

Therefore, the value of x is 30.

in a diagram of a bridge , m angle 1 = 87. If angle 1 and angle 3 are verttical angles, and angle 2 and angle 3 are suplementary, what equation can you write to solve for the value for the value of m angle 2? Let m angle 2 = x

Given that angle 1 and angle 3 are vertical angles, they are congruent. So, m angle 1 = m angle 3 = 87.

Also, it is given that angle 2 and angle 3 are supplementary, meaning their measures add up to 180 degrees. Therefore, we can write the equation:

m angle 2 + m angle 3 = 180

Substitute the values we have:

x + 87 = 180

Now, solve for x:

x = 180 - 87
x = 93

Therefore, the value of m Angle 2 (x) is 93.

its incorrect...